Absolute Max/Min of a Trig Function

Click For Summary
To find the maximum and minimum values of a trigonometric function over a specified interval, first calculate the derivative and set it to zero to identify critical points. In the example provided, the function f(t) = 2cos(t) + sin(2t) leads to the derivative f'(t) = 2cos(2t) - 2sin(t). The equation cos(2t) - sin(t) = 0 can be simplified using the identity cos(2t) = 1 - 2sin²(t), transforming it into a quadratic equation in terms of sin(t). Solving this quadratic will yield the critical points needed to determine the maximum and minimum values of the function within the given interval. This approach effectively combines calculus with trigonometric identities to solve the problem.
Sczisnad
Messages
9
Reaction score
0
I am having some issues with going about finding the max and min of trig functions over a set interval. Normally with finding the max and min of a function over an interval the first thing that I do is find the derivative of that function, Then i set the derivative to 0 and solve to find the critical points. Now this is where I run into problems...

Ex. Find the Max/Min values of f on the interval [0,pi/2] f(t)=2cos(t)+sin(2t)My work:

f(t)=2cos(t)+sin(2t) "the original function"

f'(t)=2cos(2t)-2sin(t) "I found it's derivative"

0=2cos(2t)-2sin(t) "finding where f' is equal to 0"

0=cos(2t)-sin(t) "simplified"

But now what? With a normal function you get a nice x=?,?,? ...

SO how can i find the critical values for a trig function and how can I then find the Max and Min over a set interval?
 
Physics news on Phys.org
You can use trig identities to rewrite \cos(2t) entirely in terms of \sin^2t. This will leave you with a quadratic equation in terms of \sin t to solve.
 
So you need to solve cos(2t)-sin(t)=0. Is suggest you first use the formula cos(2t)=1-2sin²(t). Then you have a quadratic equation in sin(t) which can be solved with the usual methods...
 
Question: A clock's minute hand has length 4 and its hour hand has length 3. What is the distance between the tips at the moment when it is increasing most rapidly?(Putnam Exam Question) Answer: Making assumption that both the hands moves at constant angular velocities, the answer is ## \sqrt{7} .## But don't you think this assumption is somewhat doubtful and wrong?

Similar threads

  • · Replies 3 ·
Replies
3
Views
2K
  • · Replies 30 ·
2
Replies
30
Views
3K
  • · Replies 12 ·
Replies
12
Views
2K
  • · Replies 5 ·
Replies
5
Views
2K
  • · Replies 9 ·
Replies
9
Views
2K
Replies
1
Views
3K
  • · Replies 8 ·
Replies
8
Views
2K
Replies
2
Views
2K
  • · Replies 3 ·
Replies
3
Views
2K
  • · Replies 6 ·
Replies
6
Views
2K